If u and v are positive real numbers, is u>v?

This topic has expert replies
Legendary Member
Posts: 712
Joined: Fri Sep 25, 2015 4:39 am
Thanked: 14 times
Followed by:5 members

If u and v are positive real numbers, is u>v?

by Mo2men » Mon Mar 26, 2018 1:45 am
If u and v are positive real numbers, is u>v?

1. u^3/v < 1

2. (u^1/3) /v < 1

OA: C
Source: Manhattan

[spoiler]Any short cut when combine both statements?[/spoiler]

User avatar
GMAT Instructor
Posts: 15539
Joined: Tue May 25, 2010 12:04 pm
Location: New York, NY
Thanked: 13060 times
Followed by:1906 members
GMAT Score:790

u and v

by GMATGuruNY » Mon Mar 26, 2018 2:47 am
Mo2men wrote:If u and v are positive real numbers, is u>v?

1. u^3/v < 1

2. (u^1/3) /v < 1
Since u and v are POSITIVE, the inequalities in the statements can be simplified by multiplying each side by v.

Statement 1, rephrased: u³ < v
Case 1: u=1 and v=2
In this case, u < v, so the answer to the question stem is NO.
Case 2: u=1/2 and v=1/3
In this case, u > v, so the answer to the question stem is YES.
INSUFFICIENT.

Statement 2, rephrased: ∛u < v
Case 1: u=1 and v=2
In this case, u < v, so the answer to the question stem is NO.
Case 3: u=8 and y=7
In this case, u > v, so the answer to the question stem is YES.
INSUFFICIENT.

Statements combined:
Test whether it's possible for both statements to be satisfied if v < u.
Adding v < u to Statement 1 yields the following:
u³ + v < u + v
u³ < u.
Here, u must be a FRACTION.
Adding v < u to Statement 2 yields the following:
∛u + v < u + v
∛u < u.
Here, u must be GREATER THAN 1.
Since it is not possible for u simultaneously to be both a fraction and a value greater than 1, the two statements cannot both be satisfied if v < u.
Implication:
For both statements to be satisfied, u must NOT be greater than v.
Thus. the answer to the question stem is NO.
SUFFICIENT.

The correct answer is C.
Last edited by GMATGuruNY on Mon Mar 26, 2018 12:05 pm, edited 1 time in total.
Private tutor exclusively for the GMAT and GRE, with over 20 years of experience.
Followed here and elsewhere by over 1900 test-takers.
I have worked with students based in the US, Australia, Taiwan, China, Tajikistan, Kuwait, Saudi Arabia -- a long list of countries.
My students have been admitted to HBS, CBS, Tuck, Yale, Stern, Fuqua -- a long list of top programs.

As a tutor, I don't simply teach you how I would approach problems.
I unlock the best way for YOU to solve problems.

For more information, please email me (Mitch Hunt) at [email protected].
Student Review #1
Student Review #2
Student Review #3

Legendary Member
Posts: 712
Joined: Fri Sep 25, 2015 4:39 am
Thanked: 14 times
Followed by:5 members

If u and v are positive real numbers, is u>v?

by Mo2men » Mon Mar 26, 2018 3:27 am
GMATGuruNY wrote:
Statements combined:
Test whether it's possible for both statements to be satisfied if v < u.
Adding v < u to Statement 1 yields the following:
u³ + v < u + v
u³ < u.
Here, u must be a FRACTION.
Adding v < u to Statement 2 yields the following:
∛u + v < u + v
∛u < u.
Here, u must be GREATER THAN 1.
Since it is not possible for u to be both a fraction and a value greater than 1, the two statements cannot both be satisfied if v < u.
Implication:
For both statements to be satisfied, u must NOT be greater than v.
Thus. the answer to the question stem is NO.
SUFFICIENT.
The correct answer is C.
Thanks Mitch for you support,

I have general question to understand better the method that you used to solve the problem.

1- is not it incorrect to jump to the conclusion based on testing one condition ( v < u)?

2-You have test that v < u , what if the result of your test proved that v < u? Should I test if v > u to cover all situations? or can I conclude directly that u >v?

Thanks

User avatar
GMAT Instructor
Posts: 15539
Joined: Tue May 25, 2010 12:04 pm
Location: New York, NY
Thanked: 13060 times
Followed by:1906 members
GMAT Score:790

by GMATGuruNY » Mon Mar 26, 2018 4:43 am
Mo2men wrote:Thanks Mitch for you support,

I have general question to understand better the method that you used to solve the problem.

1- is not it incorrect to jump to the conclusion based on testing one condition ( v < u)?
The question stem asks whether v < u.
As shown in my solution above, v < u would require that u simultaneously be both a fraction and a value greater than 1.
Since it is not possible for u to be simultaneously both a fraction and a value greater than 1, we can conclude that v is NOT less than u.
Thus, the answer to the question stem is NO.
2-You have test that v < u , what if the result of your test proved that v < u? Should I test if v > u to cover all situations? or can I conclude directly that u >v?
If we were able to show that v < u is possible, we would then test whether u < v is also possible.
If both cases were possible, then the correct answer would be E.
Private tutor exclusively for the GMAT and GRE, with over 20 years of experience.
Followed here and elsewhere by over 1900 test-takers.
I have worked with students based in the US, Australia, Taiwan, China, Tajikistan, Kuwait, Saudi Arabia -- a long list of countries.
My students have been admitted to HBS, CBS, Tuck, Yale, Stern, Fuqua -- a long list of top programs.

As a tutor, I don't simply teach you how I would approach problems.
I unlock the best way for YOU to solve problems.

For more information, please email me (Mitch Hunt) at [email protected].
Student Review #1
Student Review #2
Student Review #3